Sunday, July 15, 2018

How To Do A Set Of Logic Games

It's come to my attention that the Logic Games (or Analytical Reasoning) section of the LSAT is pretty difficult for most people. I had the good fortune of getting a book of just logic games a few months before I started actually prepping for the LSAT, and I drilled them enough so that they're my best section. 

There's a simple enough formula to answering most of the questions, and I'll take you through it step-by-step below. The sample problem I'm using is from the REA 2nd Edition Logic Games book (found on Amazon here). It's not endorsed by the LSAC (Law School Admission's Council), and in my opinion, the questions can be harder than the ones you see on the LSAT. That's not necessarily a bad thing, though; when you're prepping for a test you might prefer to study to a higher level than what you'll need to do, so that the test itself feels a little easier. 

Here's a set up for the rules of the set:



So the first thing to do is to set up a diagram based off of what we've learned in the question. Each school will be ranked 1-5 in each category, and no school can have the same rank in a category, so we can set up a chart that explains what that means. 



Next, we need to write out the rules in short hand, so that they can all be easily referred to during the test. There are some that affect the chart no matter what, and are now set, so we can go ahead and mark those in.



The other rules are about order, and don't let the different wording confuse you. They're saying the same thing. It doesn't matter what symbols that you choose to use to represent the rules, it only matters that you'll be able to remember them.



Next, we move on to the questions. Question #1:

(Note: Question #1 reads in full: Which of the following is a pair that can have no number 1 rankings?)

This one is pretty simple. We know that O has to be ranked below W at all times, so O cannot be ranked 1 in any categories. We also know that A is fixed in 4 and 3 in terms of Cost and Location, and W is 1 in Quality, so A cannot be ranked 1 either. Therefore, the answer is D.

This is why you want to be familiar with the rules and have your diagram written out; otherwise you'd have to go through each answer and see whether or not it was possible for each option to be ranked 1.

Question #2:



This question is trying to trick you, but if we look back at the rules, we'll realize that the condition about Ts ranking is superfluous. W must always be ranked higher than O. It doesn't matter where T is. W cannot be ranked 5. Therefore, the correct answer is E.

Again, have your rules written out in shorthand, right by you. All it takes is a quick glance to remember that W must always be higher than O.

Question #3:




This is a tough one, because it really just requires us to plug in every option and see whether or not it works. There's no way to get around these, except to be quick and have the rules close so that we can keep referring to them.

This first diagram proves that W can rank 2 in Cost, so A is not correct.



This second diagram proves that W can rank 2 in Location, so B is not correct. 


These next diagrams proves that O cannot rank 5 in Quality.



Because of the rule about O having to be lower than W, and the positions of the fixed As, either the O in Cost or Location must be in position five, and since no school can have the same rank in different categories, the O in Quality cannot be ranked 5. Therefore, the correct answer is C.

Once we've found the correct answer, we can move on to the next question. It can be handy to keep the diagrams we've already done close by, in case we need them for another question.

Question #4:



Now that we've got a conditional, we need to diagram out that conditional. But we can automatically strike answer A, since we know from Question #3 that O cannot be ranked 5 in Quality.

The tricky thing about these conditionals is that they don't ask which could be true, only which must be true. So the first thing to do is fill in what we know has to be true, given the condition. We get the following as our answer. 



When we compare that diagram, which must be true, to the options available, we see that O must be ranked 3 in Costs. Therefore, the correct answer must be C.

Question #5:


(Note: Question #5 reads in full: If S is ranked 2 in location, which of the following must be ranked 2 in quality?)

Another conditional. Based off of this, we can fill in the whole diagram:



Because of the condition on the position of S, and the rules we were given at the beginning of the set, we can see that O must be ranked 2 in Quality. Therefore, the correct answer must be D.

Question #6:



As with the last question, we'll fill in what we have so far. Be careful with the wording, though; this question doesn't ask what must be true, only what could be. Here's what we can deduce from the question:



It's simple to go through and eliminate what we know to be false because of this. 

O cannot be ranked 4 in Quality since it is already ranked 4 in Location, so B is incorrect. W cannot be ranked 3 in Location, since it is already ranked 2 in Location, so C is incorrect. T could not be ranked 3 in Cost, since W is ranked 3 in Cost, so D is incorrect. A cannot be ranked 4 in Quality, since it is already ranked 4 in Cost, so E is incorrect. There is only one answer remaining. Therefore, the correct answer must be A.

Question #7:



The first thing to do here is to figure out what the board looks like if these conditions are met. We can determine the whole board from them:



The only important thing to remember here is that 'lowest' really means numerically, not in the sense that a 5 would be the lowest ranking a school could get. That being said, you add them all up and you see that:

T = 5 + 1 + 4 = 10
S = 1 + 2 + 3 = 6
A = 4 + 3 + 5 = 12
O = 2 + 3 + 5 = 10
W = 1 + 2 + 4 = 7

So the school with the lowest score is S. Therefore, the correct answer is B.

I know it seems like a ridiculously frustrating method, but with practice, these questions really do get easier. You just have to remember to simplify the rules, build your set-up diagram, and mark the constants that are true no matter the question. Keep in mind that the questions are designed to trick you, so always look for the simplest solution first.

No comments:

Post a Comment